ChaseDream
搜索
返回列表 发新帖
查看: 17112|回复: 50
打印 上一主题 下一主题

[逻辑小分队] 【每日逻辑练习第二季】【1-17】

[复制链接]
跳转到指定楼层
楼主
发表于 2011-12-5 19:35:35 | 只看该作者 回帖奖励 |倒序浏览 |阅读模式
今日作业哈~
【精练】
4. Medical researcher: As expected, records covering
the last four years of ten major hospitals
indicate that babies born prematurely were
more likely to have low birth weights and to
suffer from health problems than were babies
not born prematurely. These records also
indicate that mothers who had received
adequate prenatal care were less likely to have
low birth weight babies than were mothers who
had received inadequate prenatal care.
Adequate prenatal care, therefore, significantly
decreases the risk of low birth weight babies.
Which one of the following, if true, most weakens the
medical researcher’s argument?
(A) The hospital records indicate that many babies
that are born with normal birth weights are
born to mothers who had inadequate prenatal
care.
(B) Mothers giving birth prematurely are routinely
classified by hospitals as having received
inadequate prenatal care when the record of
that care is not available.
(C) The hospital records indicate that low birth
weight babies were routinely classified as
having been born prematurely.
(D) Some babies not born prematurely, whose
mothers received adequate prenatal care,
have low birth weights.
(E) Women who receive adequate prenatal care are
less likely to give birth prematurely than are
women who do not receive adequate prenatal
care.

【逻辑链】
65. (35041-!-item-!-188;#058&007595)
Sales of telephones have increased dramatically over the last year. In order to take
advantage of this increase, Mammoth Industries plans to expand production of its own model
of telephone, while continuing its already very extensive advertising of this product.
Which of the following, if true, provides most support for the view that Mammoth Industries
cannot increase its sales of telephones by adopting the plan outlined above?
(A) Although it sells all of the telephones that it produces, Mammoth Industries' share of all
telephone sales has declined over the last year.
(B) Mammoth Industries' average inventory of telephones awaiting shipment to retailers has
declined slightly over the last year.
(C) Advertising has made the brand name of Mammoth Industries' telephones widely known,
but few consumers know that Mammoth Industries owns this brand.
(D) Mammoth Industries' telephone is one of three brands of telephone that have together
accounted for the bulk of the last year's increase in sales.
(E) Despite a slight decline in the retail price, sales of Mammoth Industries' telephones have
fallen in the last year.


66. (35089-!-item-!-188;#058&007596)
Bank depositors in the United States are all financially protected against bank failure because
the government insures all individuals' bank deposits. An economist argues that this
insurance is partly responsible for the high rate of bank failures, since it removes from
depositors any financial incentive to find out whether the bank that holds their money is
secure against failure. If depositors were more selective, then banks would need to be
secure in order to compete for depositors' money.
The economist's argument makes which of the following assumptions?
(A) Bank failures are caused when big borrowers default on loan repayments.
(B) A significant proportion of depositors maintain accounts at several different banks.
(C) The more a depositor has to deposit, the more careful he or she tends to be in selecting
a bank.
(D) The difference in the interest rates paid to depositors by different banks is not a
significant factor in bank failures.
(E) Potential depositors are able to determine which banks are secure against failure.


67. (35137-!-item-!-188;#058&007600)
A drug that is highly effective in treating many types of infection can, at present, be obtained
only from the bark of the ibora, a tree that is quite rare in the wild. It takes the bark of
5,000 trees to make one kilogram of the drug. It follows, therefore, that continued
production of the drug must inevitably lead to the ibora's extinction.
Which of the following, if true, most seriously weakens the argument above?
(A) The drug made from ibora bark is dispensed to doctors from a central authority.
(B) The drug made from ibora bark is expensive to produce.
(C) The leaves of the ibora are used in a number of medical products.
(D) The ibora can be propagated from cuttings and grown under cultivation.
(E) The ibora generally grows in largely inaccessible places.


68. (35185-!-item-!-188;#058&007601)
Manufacturers sometimes discount the price of a product to retailers for a promotion period
when the product is advertised to consumers. Such promotions often result in a dramatic
increase in amount of product sold by the manufacturers to retailers. Nevertheless, the
manufacturers could often make more profit by not holding the promotions.
Which of the following, if true, most strongly supports the claim above about the
manufacturers' profit?
(A) The amount of discount generally offered by manufacturers to retailers is carefully
calculated to represent the minimum needed to draw consumers' attention to the product.
(B) For many consumer products the period of advertising discounted prices to consumers is
about a week, not sufficiently long for consumers to become used to the sale price.
(C) For products that are not newly introduced, the purpose of such promotions is to keep
the products in the minds of consumers and to attract consumers who are currently using
competing products.
(D) During such a promotion retailers tend to accumulate in their warehouses inventory
bought at discount; they then sell much of it later at their regular price.
(E) If a manufacturer fails to offer such promotions but its competitor offers them, that
competitor will tend to attract consumers away from the manufacturer's product.
收藏收藏 收藏收藏
沙发
 楼主| 发表于 2011-12-5 19:36:14 | 只看该作者
逻辑链答案:EEDD
精练解析:
Question #4. Weaken-CE. October 1999 LSAT, Section 2, #24. The correct answer choice is (B)
The premises contain correlations, and the conclusion makes a causal claim:
PC = adequate prenatal care
DR = decrease risk of low birth weight babies
C          E
PC →  DR
The question stem asks you to weaken the argument, and the correct answer falls into one of the five basic
methods for weakening a causal argument.
Answer choice (A): The conclusion specifically states that mothers who had received adequate prenatal
care were less likely to have low birth weight babies than mothers who had received inadequate prenatal
care. Thus, although mothers who received inadequate prenatal care have a higher likelihood of having
low birth weight babies, this likelihood still allows for many babies to be born of normal weight. In a later
chapter we will explore the ways the LSAT uses numbers and statistics to confuse test takers, but for now,
consider this analogy: The Detroit Tigers are more likely to lose a baseball game than any other team, but
even so, they can still win a number of games. In the same way, the aforementioned mothers may be more
likely to have low birth weight babies, but they can still give birth to babies of normal weight. Hence,
answer choice (A) does not attack the argument.
Answer choice (B): This is the correct answer. The answer choice falls into the category of “Showing a
statistical problem exists with the data used to make the causal statement.” By indicating that all mothers
without prenatal care records are automatically classified as mothers receiving inadequate prenatal care, the
answer undermines the relationship in the argument because the data used to make the conclusion is
unreliable.
Answer choice (C): The conclusion is about low birth weight babies, not premature babies. Even if low
birth weight babies were routinely classified as premature, that would not affect the conclusion.
Answer choice (D): Similar to answer choice (A), the likelihoods discussed in the stimulus allow for this
possibility. Hence, this answer cannot hurt the argument.
Answer choice (E): If anything, this answer strengthens the argument since it shows that adequate prenatal
care has a powerful positive effect.
板凳
发表于 2011-12-5 22:13:40 | 只看该作者
沙发,坐稳~~

---------------------------------------------------
精练--------------------42s----------------------weakenP: The baby born non-prematurely would suffer low weight than baby born prematurely.
P: The women who don't have adequate prenatal care would have low weight baby than those who have adequate prenatal care.
C: Therefore, adequate prenatal care can prevent risk of low weight baby.
Prephrase: Women with adequate prenatal care can still have low weight baby because of other accidents.
Analysis:
(A) The hospital records indicate that many babies
that are born with normal birth weights are
born to mothers who had inadequate prenatal
care.----------------------------------------------------------this can be the contender, but the argument does not state that mothers with inadequate prenatal cannot have normal birth weight babies. 它只说more likely。
(B) Mothers giving birth prematurely are routinely
classified by hospitals as having received
inadequate prenatal care when the record of
that care is not available.---------------------------------This works to weaken the conclusion, since mothers who have normal weight babies are actually justified to receive inadequate care.
(C) The hospital records indicate that low birth
weight babies were routinely classified as
having been born prematurely.---------------------------This only mentions baby itself, it doesn't help to weaken the conclusion.
(D) Some babies not born prematurely, whose
mothers received adequate prenatal care,
have low birth weights.------------------------------------support the conclusion.
(E) Women who receive adequate prenatal care are
less likely to give birth prematurely than are
women who do not receive adequate prenatal
care.-----------------------------------------------------------A推B可以,但是非A推非B好像就不对了吧。


恩,看来我的理解是正确的,B的区别在于说的是all mothers,所以samples全部涵盖了,这样就变得更有力了。
E的选项解释看的不是很清晰阿, If anything, this answer strengthens the argument since it shows that adequate prenatal care has a powerful positive effect.好像正好说反了吧/
E明明说的是有恰当的care的妇女比没有受到恰当care的妇女更少的give birth prematurely.这样怎么会support conclusion?bible解释错了吧~~~
地板
发表于 2011-12-5 22:52:14 | 只看该作者
不知道是不是板凳呢~
5#
发表于 2011-12-5 23:21:51 | 只看该作者
1.premise: As expected, records covering the last four years of ten major hospitals indicate that babies born prematurely were more likely to have low birth weights and to suffer from health problems than were babies not born prematurely. These records also indicate that mothers who had received adequate prenatal care were less likely to have low birth weight babies than were mothers who had received inadequate prenatal care. conclusion:Adequate prenatal care, therefore, significantly decreases the risk of low birth weight babies.
babies born prematurely were more likely to have low birth weights
it's not that low birth weights babies are most likely to be prematurely born one
(A) The hospital records indicate that many babies that are born with normal birth weights are born to mothers who had inadequate prenatal care.
--???--not attack
(B) Mothers giving birth prematurely are routinely classified by hospitals as having received inadequate prenatal care when the record of that care is not available.
--right
(C) The hospital records indicate that low birth weight babies were routinely classified as having been born prematurely.
--irrelevant
(D) Some babies not born prematurely, whose mothers received adequate prenatal care, have low birth weights.
--irrelevant
(E) Women who receive adequate prenatal care are less likely to give birth prematurely than are women who do not receive adequate prenatal care.
--irrelevant
好纠结的一道题2.background information:Sales of telephones have increased dramatically over the last year.
premise:Mammoth Industries plans to expand production of its own model of telephone, while continuing its already very extensive advertising of this product.
conclusion: to take advantage of this increase,
maybe people buy new products~many people have already had one
(A) Although it sells all of the telephones that it produces, Mammoth Industries' share of all telephone sales has declined over the last year.
--irrelevant
(B) Mammoth Industries' average inventory of telephones awaiting shipment to retailers has declined slightly over the last year.
--irrelevant
(C) Advertising has made the brand name of Mammoth Industries' telephones widely known, but few consumers know that Mammoth Industries owns this brand.
--right?
(D) Mammoth Industries' telephone is one of three brands of telephone that have together accounted for the bulk of the last year's increase in sales.
--irrelevant
(E) Despite a slight decline in the retail price, sales of Mammoth Industries' telephones have fallen in the last year.
--irrelevant--right
3.background information:Bank depositors in the United States are all financially protected against bank failure because the government insures all individuals' bank deposits.
premise: since it removes from depositors any financial incentive to find out whether the bank that holds their money is secure against failure. If depositors were more selective, then banks would need to be secure in order to compete for depositors' money.
conclusion: An economist argues that this insurance is partly responsible for the high rate of bank failures,
depositors have access to get the information of banks
(A) Bank failures are caused when big borrowers default on loan repayments.
--irrelevant
(B) A significant proportion of depositors maintain accounts at several different banks.
--weaken
(C) The more a depositor has to deposit, the more careful he or she tends to be in selecting a bank.
--irrelevant
(D) The difference in the interest rates paid to depositors by different banks is not a significant factor in bank failures.
--irrelevant
(E) Potential depositors are able to determine which banks are secure against failure.
--right
4. background information:A drug that is highly effective in treating many types of infection can, at present, be obtained only from the bark of the ibora, a tree that is quite rare in the wild.
premise:It takes the bark of 5,000 trees to make one kilogram of the drug.
conclusion:that continued production of the drug must inevitably lead to the ibora's extinction.
how about people can plant the trees in a certain condition?
(A) The drug made from ibora bark is dispensed to doctors from a central authority.
--irrelevant
(B) The drug made from ibora bark is expensive to produce.
--irrelevant
(C) The leaves of the ibora are used in a number of medical products.
--irrelevant
(D) The ibora can be propagated from cuttings and grown under cultivation.
--right
(E) The ibora generally grows in largely inaccessible places.
--against information offered
5.Manufacturers sometimes discount the price of a product to retailers for a promotion period when the product is advertised to consumers. Such promotions often result in a dramatic increase in amount of product sold by the manufacturers to retailers. Nevertheless, the manufacturers could often make more profit by not holding the promotions. Which of the following, if true, most strongly supports the claim above about the manufacturers' profit?
(A) The amount of discount generally offered by manufacturers to retailers is carefully calculated to represent the minimum needed to draw consumers' attention to the product.
--irrelevant
(B) For many consumer products the period of advertising discounted prices to consumers is about a week, not sufficiently long for consumers to become used to the sale price.
--right
(C) For products that are not newly introduced, the purpose of such promotions is to keep the products in the minds of consumers and to attract consumers who are currently using competing products.
--irrelevant
(D) During such a promotion retailers tend to accumulate in their warehouses inventory bought at discount; they then sell much of it later at their regular price.
--irrelevant--right???
(E) If a manufacturer fails to offer such promotions but its competitor offers them, that competitor will tend to attract consumers away from the manufacturer's product.
--weaken
6#
发表于 2011-12-6 01:20:08 | 只看该作者
无缘无故按错键了,又占了一楼~~
7#
发表于 2011-12-6 08:09:13 | 只看该作者

background:
As expected, records covering the last four years of ten major hospitals indicate that babies born prematurely were more likely to have low birth weights and to suffer from health problems than were babies not born prematurely

Premise: These records also indicate that mothers who had received adequate prenatal care were less likely to have low birth weight babies than were mothers who had received inadequate prenatal care.

Conclusion: Adequate prenatal care, therefore, significantly decreases the risk of low birth weight babies
(A) The hospital records indicate that many babies
that are born with normal birth weights are
born to mothers who had inadequate prenatal
care.
(B) Mothers giving birth prematurely are routinely
classified by hospitals as having received
inadequate prenatal care when the record of
that care is not available.

--------------------------------------------------------correct
(C) The hospital records indicate that low birth
weight babies were routinely classified as
having been born prematurely.
(D) Some babies not born prematurely, whose
mothers received adequate prenatal care,
have low birth weights.
(E) Women who receive adequate prenatal care are
less likely to give birth prematurely than are
women who do not receive adequate prenatal
care.

Background: Bank depositors in the United States
are all financially protected against bank failure because the government insures all individuals' bank deposits
Premise: An economist argues that this insurance is partly responsible for the high rate of bank failures, since it removes from depositors any financial incentive to find out whether the bank that holds their money is secure against failure

Conclusion: If depositors were more selective, then banks would need to be secure in order to compete for depositors' money.

(A) Bank failures are caused when big borrowers default on loan repayments.

(B) A significant proportion of depositors maintain accounts at several different banks.
(C) The more a depositor has to deposit, the more careful he or she tends to be in selecting
a bank.
(D) The difference in the interest rates paid to depositors by different banks is not a
significant factor in bank failures.

----------------------------------------------------------------------------------correct
(E) Potential depositors are able to determine which banks are secure against failure.

又错了

BackgroundA drug that is highly effective in treating many types of infection can, at present, be obtained
only from the bark of the ibora, a tree that is quite rare in the wild

Conclusion: continued production of the drug must inevitably lead to the ibora's extinction.

Premise: It takes the bark of 5,000 trees to make one kilogram of the drug.

(A)The drug made from ibora bark is dispensed to doctors from a central authority.
(B) The drug made from ibora bark is expensive to produce.
(C) The leaves of the ibora are used in a number of medical products.
(D) The ibora can be propagated from cuttings and grown under cultivation.

------------------------------------------------------------------------------------------correct
(E) The ibora generally grows in largely inaccessible places.

Background: Manufacturers sometimes discount the price of a product to retailers for a promotion period
when the product is advertised to consumers

Premise: Such promotions often result in a dramatic
increase in amount of product sold by the manufacturers to retailers

conclusion the
manufacturers could often make more profit by not holding the promotions.

(A)The amount of discount generally offered by manufacturers to retailers is carefully
calculated to represent the minimum needed to draw consumers' attention to the product.
(B) For many consumer products the period of advertising discounted prices to consumers is
about a week, not sufficiently long for consumers to become used to the sale price.
(C) For products that are not newly introduced, the purpose of such promotions is to keep
the products in the minds of consumers and to attract consumers who are currently using
competing products.
(D) During such a promotion retailers tend to accumulate in their warehouses inventory
bought at discount; they then sell much of it later at their regular price.

-----------------------------------------------------------------------------------------correct
(E) If a manufacturer fails to offer such promotions but its competitor offers them, that
competitor will tend to attract consumers away from the manufacturer's product.
8#
发表于 2011-12-6 11:38:43 | 只看该作者
Weaken  
Baby prematurely born is more likely to be low weight and to suffer from problems. Mother who has adequate prenatal care is less likely to have a low weight baby than that who don’t have.
Prenatal care can significantly reduce the risk of low weight baby.
Pre:  这道题竟然看糊涂了 又看了看才明白
Adequate prenatal care---------------?less likely low weight baby
Weaken的话最好找他因导致,文中提到的就是premature
(A) The hospital records indicate that many babies     这个有点意思但是只说many不确定,
that are born with normal birth weights are           文中结论也是significantly reduce也不
born to mothers who had inadequate prenatal care.     是全部,这个weaken不好
(B) Mothers giving birth prematurely are routinely     这个是答案,早产才是low baby出现的
classified by hospitals as having received
inadequate prenatal care when the record of
that care is not available.
(C) The hospital records indicate that low birth        这个无关
weight babies were routinely classified as
having been born prematurely.
(D) Some babies not born prematurely, whose       这个不充分,文中本来都的是不是完全确定口气,some显得太薄弱
mothers received adequate prenatal care,
have low birth weights.
(E) Women who receive adequate prenatal care are    这里是说care和早不早产有关,
less likely to give birth prematurely than are
women who do not receive adequate prenatal
care.
9#
发表于 2011-12-6 17:15:38 | 只看该作者
1/Background: babies born prematurely are more likely to have less birth weight and catch health problems than babies not born prematurely. Mothers who had received adequate prenatal care were less likely to have low birth weight babies than were mothers who had received inadequate prenatal care.
Premise: adequate prenatal care significantly decrease the risk of low birth weight babies.
Prephrase: adequate prenatal care is hardly effective to the prematurely born babies.
B
A: even it is true, it cannot be presumed that the adequate prenatal care is useless.
B: when the mothers giving birth prematurely are classified as having received inadequate prenatal care, the base number of low birth weight babies increases, then make the adequate prenatal care look useful.
C: irrelevant
D: only some babies, not sufficient to attack the argument
E: irrelevant
2/Background: sales of telephones have increased dramatically over the last year.
Premise: Mammoth plans to expand production of its model of telephone and continue the already very expensive advertising of the product.
Prephrase: the sales of Mammoth’s telephone have continued to decline.
E
A: market share decline does not mean the sales decline
B: irrelevant
C: irrelevant
D: it supports the argument, not weakens.
E: the fallen in sales had showed that consumers had less interest in Mammoth’s telephones, if it continues the production and the advertising, it will suffer more loss.
3/Background: as the government insures all individuals’ bank deposits, bank depositors are all protected against bank failure.
Premise: the government’s insurance should be partly responsible for the failure of banks, because the insurance for depositors make the competition less among banks.
Prephrase: being capable of holding deposits secure against failure is a important standard when depositors choose banks.
E
A: irrelevant
B: irrelevant
C: nothing about “against failure”
D: irrelevant
E: if depositors can determine which banks are secure, then the supposed unsecure bank will improve
4/Background: it takes 5,000 trees to make a highly effective drug
Premise: since the trees are rare in the wild, continuing the production of the drug will inevitably lead to the extinction of the trees.
Prephrase: as the drug is high profitable, the trees have been planted in a large area.
D
A: irrelevant
B: irrelevant
C: irrelevant
D: the ibora has strong breed ability and can be cultivated artificially.
E: irrelevant
5/Background: the promotion of reducing price of products often causes a dramatic increase in sales.
Premise: the manufacturers could often make more profit by not holding the promotions.
Prephrase: the retailers got large amoun of products from the manufacturers during the period of promotion
D
A: irrelevant
B: irrelevant
C: irrelevant
D: more inventory of the retailers, more sales of the manufacturers
E: irrelevant
10#
发表于 2011-12-6 17:58:19 | 只看该作者
精炼题
50s-weaken
The research show that babies who born prematurely were more likely to have low birth weights. Mothers who have received adequate prenatal care are less likely to have low birth weight babies.Therefore, adequate prenatal care significantly decreases the risk of low birth weight babies.
Prephase: The data is not accurate

(A) The hospital records indicate that many babies that are born with normal birth weights are born to mothers who had inadequate prenatal care. —not attack the causal relation (mothers who had inadequate are also likely to have a babie with normal birth weight)
(B) Mothers giving birth prematurely are routinely classified by hospitals as having received inadequate prenatal care when the record of that care is not available. -bingo
(C) The hospital records indicate that low birth weight babies were routinely classified as having been born prematurely.
-irrelevant
(D) Some babies not born prematurely, whose mothers received adequate prenatal care, have low birth weights.
-The stimulus allows for this possibility.
(E) Women who receive adequate prenatal care are less likely to give birth prematurely than are women who do not receive adequate prenatal care.-strengthen

您需要登录后才可以回帖 登录 | 立即注册

Mark一下! 看一下! 顶楼主! 感谢分享! 快速回复:

手机版|ChaseDream|GMT+8, 2024-4-26 13:06
京公网安备11010202008513号 京ICP证101109号 京ICP备12012021号

ChaseDream 论坛

© 2003-2023 ChaseDream.com. All Rights Reserved.

返回顶部